Which one of the following most accurately expresses the main point of the passage?

Ryan-Mahabir on October 24, 2019

Why is D correct? Why is B incorrect?

Thanks

Reply
Create a free account to read and take part in forum discussions.

Already have an account? log in

BenMingov on November 12, 2019

Hi Ryan, thanks for the question!

Answer choice D is correct because it encapsulates the entire passage. It discusses the main issue at hand, namely the concerns of scientists regarding the effect of patents on their research, and then incorporates the author's conclusion that this viewpoint is misguided because patents do not prevent basic research and if anything, it will help research as patents provide a monetary incentive to innovate.

Answer choice B is true. But there are two points to be careful about. There is no mention of patents, so even though it refers to market-driven, patents are still crucial to the passage. And lastly, it does not touch on the entire idea that the author is presenting about the concerns regarding patents being misguided.

Hope this helps!